User avatar
 
ManhattanPrepLSAT1
Thanks Received: 1909
Atticus Finch
Atticus Finch
 
Posts: 2851
Joined: October 07th, 2009
 
This post thanked 4 times.
 
 

Re: Q9 - (We do not need to stop...)

by ManhattanPrepLSAT1 Fri Dec 31, 1999 8:00 pm

tamwaiman Wrote:Since Rifka concludes that they don't need to stop and ask, she implies that they are not lost.
However, Craig indicates the fact is that they are lost (deny her premise) and need to stop (opposite to Rifka's conclusion).


Nice work tamwaiman! That's exactly it, though I'd adjust one word to make things a bit more clear. Rather saying that Rifka "implies" they are not lost. I think I would say that Rifka "assumes" that they are not lost.

Rifka's argument

SA ---> L

----------
~SA

assumes that they are not lost

SA ---> L
(~L)
------------
~SA

This assumption would trigger the contrapositive of the conditional relationship and would allow Rifka's conclusion to be properly drawn. So when Craig says that they are lost, and that is why they need to stop and ask for directions

L
---------
SA

Craig's evidence denies Rifka's assumption (unstated/implicit premise) that they are not lost. And nowhere in Craig's argument is it conceded that if one stops and asks for directions, then one is necessarily lost. So we cannot say that Craig accepts Rifka's evidence - it's just not addressed. So answer choice (B) does a better job of describing the way in which Craig's response relates to Rifka's argument.

(A) is false. A reason is offered.
(C) is false. The evidence is never accepted.
(D) is false. Craig's response does not offer a counter example to a general claim, but offers a different conclusion without being any more or less general than Rifka's original statement.
(E) is wrong in both respects. The evidence is not accepted and the conclusion is most definitely challenged.


#officialexplanation
 
n.v.igietseme
Thanks Received: 0
Vinny Gambini
Vinny Gambini
 
Posts: 2
Joined: January 03rd, 2011
 
 
 

Q9 - We do not need to stop

by n.v.igietseme Thu Feb 10, 2011 12:11 pm

I'm not sure I understand the difference between 'implying that an argument is invalid by accepting the truth of its premises while rejecting its conclusion" (C) and 'denying one of the premises and arriving a a different conclusion' (B)

The premise comes as a conditional. Does that make it two different premises? Is that why B is correct? Any help would be great!
User avatar
 
tamwaiman
Thanks Received: 26
Forum Guests
 
Posts: 142
Joined: April 21st, 2010
 
This post thanked 3 times.
 
trophy
Most Thankful
 

Re: Q9 - We do not need to stop

by tamwaiman Wed Jun 15, 2011 2:13 am

SA: stop & ask for directions
L: lost

Rifka: SA --> L (conditional relationship)
Craig: L --> SA (causal relationship)

Since Rifka concludes that they don't need to stop and ask, she implies that they are not lost.
However, Craig indicates the fact is that they are lost (deny her premise) and need to stop (opposite to Rifka's conclusion).
 
tplan21
Thanks Received: 5
Forum Guests
 
Posts: 13
Joined: January 20th, 2012
 
This post thanked 1 time.
 
trophy
Most Thankful
 

Re: Q9 - (We do not need to stop...)

by tplan21 Fri Jan 20, 2012 2:31 pm

I dont necessarily think Rifka is saying that she implies they are not lost.

I picked C too, but now realize what they meant:
SA --> L
negate this and it says what Rifka says:
-L --> -SA

Craig then says:
L --> SA

Craig denied (L) Rifka's premise (-L), thereby arriving at a different conclusion (SA).

Answer choice C states: Craig accepts premise (-L) and rejects conclusion (-SA), i.e. we need to SA. He doesnt say we are not lost so lets stop and ask.

We can even use the Rifka's initial statement: SA --> L
Craig accepts premise (SA) and rejects conclusion (L), i.e. we are not lost.

Now if only i thought of this when i was taking the practice test and if only i could think of it in 30secs instead of taking eternity. PS I hope i'm right on this. I'm not briliiant like most of these people here.


tamwaiman Wrote:SA: stop & ask for directions
L: lost

Rifka: SA --> L (conditional relationship)
Craig: L --> SA (causal relationship)

Since Rifka concludes that they don't need to stop and ask, she implies that they are not lost.
However, Craig indicates the fact is that they are lost (deny her premise) and need to stop (opposite to Rifka's conclusion).
 
timmydoeslsat
Thanks Received: 887
Atticus Finch
Atticus Finch
 
Posts: 1136
Joined: June 20th, 2011
 
 
trophy
Most Thanked
trophy
First Responder
 

Re: Q9 - (We do not need to stop...)

by timmydoeslsat Fri Jan 20, 2012 2:54 pm

tplan21 Wrote:I dont necessarily think Rifka is saying that she implies they are not lost.

I agree with everything you posted about this question except this part.

The argument put forward by Rifka is as you said:

SA ---> Lost

____________
~SA

The only piece of evidence to generate that conclusion is the conditional statement. She must be assuming ~Lost.
User avatar
 
ManhattanPrepLSAT1
Thanks Received: 1909
Atticus Finch
Atticus Finch
 
Posts: 2851
Joined: October 07th, 2009
 
 
 

Re: Q9 - We do not need to stop

by ManhattanPrepLSAT1 Fri Jan 20, 2012 4:55 pm

timmydoeslsat Wrote:
tplan21 Wrote:I dont necessarily think Rifka is saying that she implies they are not lost.

I agree with everything you posted about this question except this part.

The argument put forward by Rifka is as you said:

SA ---> Lost

____________
~SA

The only piece of evidence to generate that conclusion is the conditional statement. She must be assuming ~Lost.

Great discussion here! I think you're both right! While Rifka's not saying that they're not lost, she is assuming it!
 
tplan21
Thanks Received: 5
Forum Guests
 
Posts: 13
Joined: January 20th, 2012
 
 
trophy
Most Thankful
 

Re: Q9 - We do not need to stop

by tplan21 Sat Jan 21, 2012 11:45 pm

I actually didnt miss that point. I refer to it in this statement:

negate this and it says what Rifka says:
-L --> -SA
 
timmydoeslsat
Thanks Received: 887
Atticus Finch
Atticus Finch
 
Posts: 1136
Joined: June 20th, 2011
 
 
trophy
Most Thanked
trophy
First Responder
 

Re: Q9 - We do not need to stop

by timmydoeslsat Sun Jan 22, 2012 12:04 am

Yes, but the ~L part is assumed.

The ~L in the sufficient condition is not a premise of ~L.

~L ---> ~ SA

The above is the entire premise.

For the conclusion of ~SA to exist, there is an assumption of ~L.

As of now, we do not have a premise of ~L. We have a conditional with it, but we do not have ~L. It is assumed.
 
nflamel69
Thanks Received: 16
Atticus Finch
Atticus Finch
 
Posts: 162
Joined: February 07th, 2011
 
 
 

Re: Q9 - We do not need to stop

by nflamel69 Mon Sep 17, 2012 5:42 pm

Just wondering. Is Craig's argument valid? Since he didnt dispute the S + A -> L premise. Can he conclude that L -> S ? wouldn't this be a logical fallacy?
 
marcus.v.p.
Thanks Received: 0
Vinny Gambini
Vinny Gambini
 
Posts: 19
Joined: February 08th, 2013
 
 
 

Re: Q9 - We do not need to stop

by marcus.v.p. Wed Sep 04, 2013 1:41 pm

Hi,

I have a question. How do we know Rifka's necessary condition is: they are lost? What part of Rifka's statement are you getting this from?

Thanks,

Marcus
 
amil91
Thanks Received: 5
Elle Woods
Elle Woods
 
Posts: 59
Joined: August 02nd, 2013
 
This post thanked 1 time.
 
 

Re: Q9 - We do not need to stop

by amil91 Fri Nov 22, 2013 4:49 pm

marcus.v.p. Wrote:Hi,

I have a question. How do we know Rifka's necessary condition is: they are lost? What part of Rifka's statement are you getting this from?

Thanks,

Marcus

Unless is a conditional trigger. I think of unless as meaning if not. So in this instance, the conditional version of Rifka's statement is If NOT lost then we do NOT need to stop for directions. Which the contrapositive of this is If we need to stop for directions then we are lost.
 
donghai819
Thanks Received: 7
Elle Woods
Elle Woods
 
Posts: 65
Joined: September 25th, 2015
 
 
 

Re: Q9 - We do not need to stop

by donghai819 Fri May 20, 2016 4:17 pm

I got this one. C is wrong for an obvious reason.

Notation: need to stop and ask for direction = SA; Lost = Lost;

What Rifka says is:
Premise: SA -> Lost, aka, we need to stop and ask only if we are lost.
Conclusion: ~SA, aka, we don't need to stop and ask.
Her assumption: ~Lost, coz the contrapositive of the premise: ~Lost -> ~SA.

What Craig says is:
Premise: Lost -> SA.
Conclusion (implied): We do need to stop and ask.

Look, Rifka's premise is SA -> Lost, whereas Craig's is Lost -> SA, so the two have different premises; in other words, Craig refutes Rifka's premise. In addition to that, Craig thinks they should stop and ask, while Rifka doesn't think so.

So Craig doesn't accept Rifka's premise. In fact, he holds a different premise.

To sum up, Craig disagrees Rifka's premise and has his own, as SA -> Lost (Rifka) and Lost -> SA; and Craig also holds a different conclusion saying they should stop and ask.

This is exactly what B suggests.
 
abrenza123
Thanks Received: 0
Jackie Chiles
Jackie Chiles
 
Posts: 39
Joined: August 14th, 2015
 
 
 

Re: Q9 - We do not need to stop

by abrenza123 Fri Sep 06, 2019 10:44 am

I thought that B was the best way to describe Craig's argument, but for A, I was a little unclear about not offering any reason to reject the implicit premises.

Rifka has one implicit premise of ~L or not lost, correct? the SA --> L is explicit in her second sentence. Was a bit unsure of the "any implicit premises" language.

Craig denies that they are not lost, but I'm not sure if he does or does not provide a reason to reject the implicit premise - to me, it seemed that he just rejected/denied it but didn't provide a reason. What would providing a reason look like?

Does an explanation like A's "not offering a reason to reject" have some ambiguity regarding whether or not Craig actually rejects the argument? Does "not offering a reason to reject" imply that you accept?

In my mind, just because you don't offer a reason to reject something doesn't mean you don't reject it, so the answer choice doesn't inform whether or not craig rejected the argument, so it isn't a "best fit" for the function of his comment.
User avatar
 
ohthatpatrick
Thanks Received: 3806
Atticus Finch
Atticus Finch
 
Posts: 4661
Joined: April 01st, 2011
 
 
 

Re: Q9 - We do not need to stop

by ohthatpatrick Thu Sep 19, 2019 2:17 pm

I think you might be adding too much complexity to the goal of the question stem. This is essentially just a "Describe" question, so our standard for right / wrong is basically,
RIGHT: what it says is a true description / it matches
WRONG: something it says is not a true description / it doesn't match

What's the difference between an explicit premise and an implicit premise?

An implicit premise was not explicitly said. It was implied. It was assumed.
"implicit premise" = necessary assumption

Did Rifka have any assumptions in her argument (or was it otherwise airtight)?

Her argument is.
PREM: If we aren't lost, then we do not need to stop and ask for directions.
CONC: We do not need to stop and ask for directions

What is she assuming?
"We aren't lost"

Craig says "it is a fact that we are lost".

Does this, as (B) says, "deny one of R's assumptions"? Sure.
Does this, as (A) says, "give no reason to reject any of R's assumptions"? Hmmmm.

If I hear from Craig that "we are lost", isn't that a reason to reject R's implicit premise that "we aren't lost"?

I think maybe you were saying "Craig does reject one of her implicit premises, but he doesn't offer a reason why we should reject it. He just rejects it. He doesn't offer support for the idea that 'we are lost'."

If (A) were descriptively accurate, it would sound more like
Craig: We need to stop and ask for directions.

That's contradicting the conclusion without offering (us, the reader) any reason to reject one of R's assumptions.

Since he says "we are lost", we (the reader) to have a reason to reject one of R's assumptions. It's not a bulletproof reason, but it's a reason. When we first read R's argument, we just knew she was assuming that we're not lost. We had no reason to believe she was right or wrong about that assumption. Hearing Craig say "we are lost" gives us reason to think she could be wrong about that assumption.

Another subtlety I just noticed is that (A) says that Craig contradicts the conclusion, which would sound like "We DO need to stop and ask for directions".

Craig's actual conclusion is just "We need to stop", which isn't quite a contradiction. But it is, as (B) says, a "different conclusion".

Hope this helps.
 
JeremyK686
Thanks Received: 0
Vinny Gambini
Vinny Gambini
 
Posts: 11
Joined: July 11th, 2020
 
 
 

Re: Q9 - We do not need to stop

by JeremyK686 Sat Jul 11, 2020 6:24 am

Breakdown:
Rifka: If we’re not lost, we don’t need to ask for directions. We don’t need to stop and ask for directions.

Craig: But because we are lost, we need to stop.

Paraphrase Diagram:
RP: stop and ask for directions → we’re lost (rule)
RC: not-stop and ask for directions (prescriptive claim)
IP: not-lost (→ not-stop and ask for directions) (implied descriptive claim)

CP: we’re lost (denied implied descriptive claim)
CC: we need to stop (directions possible) (different/contradictory prescriptive claim)
IP: if we’re lost → we need to stop (and possibly figure our shit out) (implied rule)

Analysis:
Craig’s claim denies a fact that is implicated by Rifka’s logic and arrives at a different/contradictory prescriptive claim.

Answer Choices:
(A) The idea that Craig didn’t provide a reason for his prescription misstates the factual statement he asserts; his on-site observation that they are currently lost.

(B) This sounds like an inverse sort of idea. It’s weird how this says that it denies one of Rifka’s premises. It depends on how you interpret / translate accurately. Great lesson on translating / interpreting correctly. He denies Rifka claiming that they aren’t lost (they are lost) and arrives at a conclusion that’s different from Rifka’s (they should just stop). I’d feel more comfortable if this answer stated that it denied both premises.

(C) This expresses a counterexample. If this were true, Craig would have to implicate: we aren’t lost, but maybe we should stop and/or ask for direction. Craig pretty firmly asserts that they are lost. And he doesn’t necessarily think they should ask for directions as much as he thinks they need to just stop. A direct counterexample to an argument is a description of a possible world in which the premises of the argument are all true but the conclusion is false. This is pretty synonymous with accepting the truth of its premises while rejecting its conclusion.

(D) Isn’t this the same thing as above?

(E) Noncommittal about Rifka’s conclusion is a little weird. This answer is telling me that Craig agrees with Rifka’s premise but beats around a bush in terms of taking a stance for or against Rifka’s conclusion. This doesn’t seem to match Craig’s movement. He thinks they’re lost and that they must stop. He seems to be taking a very clear stance.
 
JeremyK686
Thanks Received: 0
Vinny Gambini
Vinny Gambini
 
Posts: 11
Joined: July 11th, 2020
 
 
 

Re: Q9 - We do not need to stop

by JeremyK686 Sat Jul 11, 2020 6:32 am

ohthatpatrick Wrote:
Another subtlety I just noticed is that (A) says that Craig contradicts the conclusion, which would sound like "We DO need to stop and ask for directions".

Craig's actual conclusion is just "We need to stop", which isn't quite a contradiction. But it is, as (B) says, a "different conclusion".

Hope this helps.

Ahhh nice I'd love to talk about this idea!

Do you (or any future posters) think there's a functional difference between implicit and explicit contradictions? Can an implicit contradiction still live in the world of things that 'contradict the conclusion'?

Although Craig doesn’t explicitly contradict Rifka’s conclusion, his contradiction is logically implied through his denial of Rifka’s implicit premise. Should this implicit contradiction then nonetheless entail the explicit contradiction? Craig’s different ‘Z-conclusion’ (they stop) is under the classification of the conclusion’s contradictory opposition (they stop and ask for directions). It would be impossible for them to stop to ask for directions while at the same time not stop at all.

Can't wait to see what people have to say!